Você está na página 1de 338

265. During the seventeenth century, America established the precedent of A) staying out of European wars if possible.

. B) relying totally on the British for defense. C) starting wars in Europe. D) being involved in every world war since 1688. E) fighting wars on both land and sea. Ans: D

266. The soldier and explorer whose leadership earned him the title Father of New France was A) Samuel de Champlain. B) Robert de La Salle. C) Antoine Cadillac. D) Des Moines. E) Edward Vincennes. Ans: A

267. France was finally able to join in the scramble for colonies in the New World as a result of the A) Protestant takeover of the French government. B) end of the religious wars. C) revocation of the Edict of Nantes. D) St. Bartholomew's Day Massacre. E) Seven Years' War. Ans: B

268. Government in New France (Canada) was A) almost completely autocratic. B) democratic. C) similar to that of the English colonies. D) noted for its trial by jury. E) free from the king's control. Ans: A

269. Unlike the English colonies in America, in New France A) there were no popularly elected assemblies. B) the crown refused to promote the welfare of French colonization. C) the population grew very rapidly. D) no valuable resources for exploitation existed. E) the colonists practiced religious toleration. Ans: A

270. The one valuable resource in New France was A) fish. B) gold. C) trees. D) corn. E) beavers. Ans: E

271. The coureurs de bois were A) French soldiers. B) French boatmen. C) Catholic priests. D) French farmers. E) French fur trappers. Ans: E

272. The population in Catholic New France grew very slowly because A) French peasants were not allowed to move. B) the Protestant Huguenots refused to move there. C) the French government was more concerned with its Caribbean island colonies. D) disease took a heavy toll on New France's inhabitants. E) of constant attacks by the Huron Indians. Ans: D

273. The primary economic pursuit of early settlers in New France was A) farming. B) fishing. C) mining. D) fur trapping. E) rum manufacturing. Ans: D

274. The Indians suffered from their association with the French in New France in all of the following ways except A) exclusion from the fur business. B) decimation of their numbers by the white man's diseases. C) violation of their religious beliefs. D) debauchery by the white man's alcohol. E) weakening of their traditional way of life. Ans: A

275. The Jesuit priests, despite their initial failure in gaining converts, played a vital role because A) of the many converts to Catholicism. B) of the health care. C) they made peace with the Indians. D) they encouraged the Indians to participate in the fur trade. E) of their exploration and work as geographers. Ans: E

276. The French wanted to control Louisiana because they A) liked its climate. B) wanted to keep the area unfortified. C) would then control the mouth of the Mississippi. D) feared Dutch expansion into the territory. E) saw it as a dumping ground for undesirables. Ans: C

277. French motives in the New World included the desire to A) establish agricultural communities to produce profitable staple crops. B) convert Indians to Protestantism. C) compete with Spain for an empire in America. D) provide a place for French religious dissenters to settle. E) compete with Portugal for an empire in America. Ans: C

278. The early wars between France and Britain in North America were notable for the A) large number of troops committed by both sides. B) lack of Indian participation. C) carry over of European tactics to America. D) use of primitive guerrilla warfare. E) all of the above. Ans: D

279. During a generation of peace following the 1713 Treaty of Utrecht, Britain provided its American colonies with A) a large military presence for protection. B) decades of salutary neglect. C) higher taxes passed by Parliament. D) stronger parliamentary direction. E) all of the above. Ans: B

280. The War of Jenkins's Ear was A) fought in European waters. B) a great victory for Spain. C) confined to the Caribbean Sea and Georgia . D) the event that established the policy of salutary neglect. E) a defeat for France. Ans: C

281. The War of Jenkins's Ear resulted in A) France allying itself with Britain. B) British troops being involved in every territory in North America. C) France losing its vast holdings in North America. D) the colony of Georgia fighting the Spanish to a standstill. E) all of the above. Ans: B

282. New England colonists were outraged when British diplomats returned _______________ to France in 1748. A) Hudson Bay B) Acadia C) Louisbourg D) Newfoundland E) Nova Scotia Ans: C

283. The clash between Britain and France for control of the North American continent sprang from their rivalry for control of A) Cape Breton Island. B) the Ohio River Valley. C) the Mississippi River. D) the Great Lakes. E) the St. Lawrence River. Ans: B

284. The reason France needed to control the Ohio Valley was to A) stop Spain from extending its empire. B) help win the War of Jenkins's Ear. C) stop the Indian attacks on its outposts. D) link its Canadian holdings with those of the lower Mississippi Valley. E) be able to put more of its settlers there in order to increase farm production. Ans: D

285. In his first military command in the French and Indian War, George Washington A) won a decisive and hard fought battle at Fort Duquesne. B) was defeated at Fort Necessity but was allowed to retreat. C) received strong support from the British. D) helped to force the French out of Nova Scotia. E) turned his twenty years of military experience to great success. Ans: B

286. The Seven Years' War was also known in America as A) the War of Jenkins's Ear. B) the French and Indian War. C) the War of Austrian Succession. D) King William's War. E) Queen Anne's War. Ans: B

287. In the colonial wars before 1754, Americans A) functioned as a unified fighting force. B) received more support from France than Britain. C) demonstrated an astonishing lack of unity. D) were not involved in combat. E) rarely involved Indians in the fighting. Ans: C

288. The immediate purpose of the Albany Congress of 1754 was to A) request the help of the British military. B) keep the Iroquois tribes loyal to the British. C) prevent the French from attacking American outposts. D) support George Washington's desire to head the colonial militia. E) block British efforts to take control of New York City. Ans: B

289. Unlike the first three Anglo-French wars, the Seven Years' War A) won the British territorial concessions. B) united British colonists in strong support of the mother country. C) was fought initially on the North American continent. D) did not affect American colonists' attitudes toward England. E) resulted in a stronger French presence in North America. Ans: C

290. Arrange the following events in chronological order: (A) George Washington surrenders Fort Necessity; (B) General Edward Braddock is defeated near Fort Duquesne; (C) British troops capture Louisbourg in their first significant victory of the French and Indian War; (D) General James Wolfe's army defeats Montcalm's on the Plains of Abraham. A) B, A, D, C B) A, B, C, D C) C, B, A, D D) A, C, B, D E) A, B, D, C Ans: B

291. The long-range purpose of the Albany Congress in 1754 was to A) achieve colonial unity and common defense against the French threat. B) propose independence of the colonies from Britain. C) declare war on the Iroquois tribe. D) prohibit New England and New York from trading with the French West Indies. E) gain peace with France. Ans: A

292. Benjamin Franklin's plan for colonial home rule was rejected by the individual colonies because A) it did not provide for the common defense. B) the British approved it. C) it did not seem to give enough independence to the colonies. D) they did not feel that they had been well represented at the Albany Congress. E) it placed too much power in the hands of local governments. Ans: C

293. As a result of General Braddock's defeat a few miles from Fort Duquesne, A) the British controlled the frontier. B) George Washington was left without a military command. C) the frontier from Pennsylvania to North Carolina was open to Indian attack. D) General Braddock was forced to leave the military. E) the British called off their planned invasion of Canada. Ans: C

294. The British invasion of Canada in 1756 during the Seven Years' War A) resulted in victory for Britain. B) concentrated on Quebec and Montreal. C) followed sound strategic planning. D) ended in defeat. E) resulted in British control of the St. Lawrence River. Ans: D

295. When William Pitt became prime minister during the Seven Years' War, he A) ended Parliament's practice of reimbursing the colonies for their war-related expenditures. B) ordered a full-scale assault on the French West Indies. C) relied heavily on the older, more cautious generals in the British Army. D) focused his military strategy on the capture of French Canada. E) remained popular with the wealthy but not the poor. Ans: D

296. The 1759 Battle of Quebec A) had little impact on the Seven Years' War. B) was a key turning point in Queen Anne's War. C) was a dramatic victory for the French. D) ended the war of French succession. E) ranks as one of the most significant victories in British and American history. Ans: E

297. In the peace arrangements that ended the Seven Years' War, A) France surrendered all of its territorial claims to North America. B) England turned Florida over to Spain. C) Spain ceded all of Louisiana, including New Orleans, to Britain. D) France lost all its valuable sugar islands in the West Indies. E) the British got all of Canada except Nova Scotia. Ans: A

298. As a result of the Seven Years' War, Great Britain A) gained control of Louisiana. B) became the dominant power in North America. C) annexed the island of Cuba. D) gained exclusive control of the slave trade. E) all of the above. Ans: B

299. For the American colonies, the Seven Years' War A) ended the myth of British invincibility. B) left them in need of experienced officers. C) offered the opportunity to grow closer to the British. D) gave them the opportunity finally to gain control of Mississippi. E) helped improve relations between Britain and the colonies. Ans: A

300. During the Seven Years' War, A) colonial militiamen were impressed with the seeming invincibility of the British regulars. B) British officers roundly praised the skillful fighting ability of colonial troops. C) British officials were disturbed by the lukewarm support of many colonials. D) the colonists lost confidence in their own military capability. E) all American trade with Spain and France ended. Ans: C

301. With the end of the Seven Years' War, the disunity, jealousy, and suspicion that had long existed in the American colonies A) continued without change. B) began to melt somewhat. C) finally came to a complete end. D) resulted in renewed acts of violence. E) none of the above. Ans: B

302. The disunity that existed in the colonies before the Seven Years' War can be attributed to A) the enormous distances between the colonies. B) geographical barriers like rivers. C) conflicting religions. D) varied nationalities. E) all of the above. Ans: E

303. France had to give up its vision of a North American New France when A) its fishing industry faltered. B) farming proved to be unprofitable. C) King Louis XIV died. D) it was defeated by the British in 1713 and 1763. E) it could not entice enough settlers to America. Ans: D

304. When the Acadians left Canada, they went to A) Florida. B) Louisiana. C) France. D) Nova Scotia. E) the French West Indies. Ans: B

305. The isolation of Louisiana's Cajun communities ended A) during the Civil War. B) only with the civil rights movement of the 1960s. C) with bridge building in the 1930s. D) with intermarriage to Germans, English, and Spanish. E) during the American Revolution. Ans: C

306. The primary thing that the Acadians and Quebecois believed that bound them together was their A) hatred for Spain. B) desire to return to France. C) military experience. D) exile to Louisiana. E) French language. Ans: E

307. With the British and American victory in the Seven Years' War, A) the American colonies grew closer to Britain. B) Americans now feared the Spanish. C) a new spirit of independence arose, as the French threat disappeared. D) the Indians were stopped from ever again launching a deadly attack against whites. E) the British no longer retaliated against the Indians. Ans: C

308. In a sense, the history of the United States began with the A) Revolutionary War. B) July 4, 1776 signing of the Declaration of Independence. C) Boston Tea Party. D) founding of the first colony in 1607. E) fall of Quebec and Montreal. Ans: E

309. With the defeat of Chief Pontiac and his alliance, the British decided to A) stabilize Indian-white relations. B) let the colonists assume financial responsibility for defending themselves. C) remove troops stationed in the colonies. D) enlist the aid of France to halt the Indian menace. E) open land west of the Appalachian mountains to settlement. Ans: A

310. Chief Pontiac decided to try to drive the British out of the Ohio Valley because A) the British were weak as a result of the Seven Years' War. B) the British had deliberately infected Indians with smallpox. C) of the Proclamation of 1763. D) the Indians were in a precarious position. E) the French government had promised to help. Ans: D

311. The Proclamation of 1763 was designed mainly to A) oppress the colonists. B) punish the Indians. C) show the power of Parliament. D) allow western settlement by the colonists. E) work out a fair settlement of the Indian problem. Ans: E

312. In the wake of the Proclamation of 1763 A) American colonists obeyed the law they hated. B) relations with France improved. C) relations between the American colonies and the British government improved. D) the American colonies believed their destiny had been destroyed. E) American colonists moved west, defying the Proclamation. Ans: E

313. The Proclamation of 1763 A) was warmly received by American land speculators. B) removed the Spanish and Indian menace from the colonial frontier. C) declared war on Chief Pontiac and his fierce warriors. D) prohibited colonial settlement west of the Appalachian Mountains. E) opened Canada to American settlement. Ans: D

314. Change in colonial policy by the British government that helped precipitate the American Revolution involved A) removing British troops from American soil. B) beginning a war with Spain. C) removing the majority of the British navy from American waters. D) compelling the American colonists to shoulder some of the financial costs of the empire. E) all of the above. Ans: D

315. When it came to the Revolution, it could be said that the American colonists were A) eager revolutionaries. B) up until the end wanting more than the rights of Englishmen. C) little concerned about economics. D) clearly opposed to tightening commercial bonds to the British. E) reluctant revolutionaries. Ans: E

316. In a broad sense, America was A) a revolutionary force from the day of its discovery by Europeans. B) a place that nurtured a love for Britain. C) completely dependent on Britain for economic support. D) a place where no new ideas took shape. E) none of the above. Ans: A

317. The American colonial exponents of republicanism argued that a just society depends on A) a powerful central government. B) a weak army. C) a strong aristocratic tradition. D) support for hierarchical institutions. E) the willingness of all citizens to subordinate their private interests to the common good. Ans: E

318. Republican belief held that the stability of society and the authority of the government A) rested with the legislature. B) depended on a strong hierarchical culture. C) rested with a strong monarchy. D) rested on an interdependence of all citizens. E) depended on the virtue of its citizenry. Ans: E

319. The radical whigs feared A) too much democracy. B) a written constitution. C) the arbitrary power of the monarchy. D) a too powerful parliament. E) all of the above. Ans: C

320. Mercantilists believed that A) a mother country needed to import more goods than it exported. B) power came from a small colonial empire. C) the mother country produced raw materials and colonies produced the finished product. D) a country's economic wealth could be measured by the amount of gold and silver in its treasury. E) colonies drained a country of its resources. Ans: D

321. The founding of the American colonies by the British was A) accomplished in a well-planned fashion. B) based on the high-minded aspirations of groups such as the Puritans and the Quakers. C) undertaken by the government in every case. D) undertaken in a haphazard manner. E) rarely undertaken by trading companies or religious groups. Ans: D

322. Under mercantilist doctrine, the American colonies were expected to do all of the following except A) supply Britain with raw materials not available there. B) become economically self-sufficient as soon as possible. C) furnish ships, seamen, and trade to bolster the strength of the Royal Navy. D) provide a market for British manufactured goods. E) refrain from exporting woolen cloth. Ans: B

323. The first Navigation Laws were designed to A) help colonists get the best possible price for their trade goods. B) eliminate Dutch shippers from the American carrying trade. C) foster a colonial economy that would offer healthy competition with Britain's. D) encourage agricultural experimentation in the colonies. E) support the mapping of the Atlantic trade routes. Ans: B

324. The British Parliament enacted currency legislation that was intended primarily to benefit A) Virginia tobacco planters. B) British merchants. C) New England merchants. D) backwoods farmers. E) the Crown. Ans: B

325. The British Crown's royal veto of colonial legislation A) was used frequently to overturn laws passed in colonial assemblies. B) prohibited colonists from conducting the slave trade. C) was what finally provoked the War of Independence. D) was used sparingly by the British Parliament. E) was opposed by many members of the British Parliament. Ans: D

326. Under the mercantilist system, the British government reserved the right to do all of the following regarding the American colonies except A) prevent the colonies from developing militias. B) restrict the passage of lax bankruptcy laws. C) nullify any colonial legislation deemed bad for the mercantilist system. D) restrain the colonies from printing paper currency. E) enumerate products that must be shipped to Britain. Ans: A

327. Before 1763 the Navigation Laws A) were enforced heavily in the American colonies and were very effective. B) hurt Great Britain more than the American colonies. C) were a great burden to only India. D) discouraged smuggling by American colonial merchants. E) were only loosely enforced in the American colonies. Ans: E

328. Despite the benefits of the mercantile system, the American colonists disliked it because A) it forced the South into a one-crop economy. B) it favored the northern over the southern colonies. C) it forced economic initiative on the colonists. D) it kept them in a state of perpetual economic adolescence. E) all of the above. Ans: D

329. In some ways, the Navigation Laws were a burden to certain colonists because A) northern merchants derived greater benefit from the system than did southern planters. B) those colonists were heavily taxed to help provide financing for the Royal Navy, which protected colonial and British trade. C) they stifled economic initiative. D) Britain had the only European empire based on mercantilistic principles. E) they gave greater benefits to slaves holders. Ans: C

330. A new relationship between Britain and its American colonies was initiated in 1763 when ____________________ assumed charge of colonial policy. A) Charles Townshend B) George Grenville C) Lord North D) William Pitt E) King George III Ans: B

331. Match each act below with the correct description. A. Sugar Act B. Stamp Act C. Declaratory Act 1. first British law intended to raise revenues in the colonies 2. asserted Parliament's absolute power over the colonies 3. required colonists to lodge British troops in their homes 4. generated the most protest in the colonies. A) A-3, B-2, C-l B) A-1, B-4, C-3 C) A-1, B-4, C-2 D) A-4, B-1, C-2 E) A-2, B-1, C-4 Ans: C

332. The first law ever passed by Parliament for raising tax revenues in the colonies for the crown was the A) Stamp Act. B) Declaratory Act. C) Townshend Acts. D) Quartering Act. E) Sugar Act. Ans: E

333. The British Parliament passed the Stamp Act to A) raise money to support new military forces needed for colonial defense. B) punish the American colonists. C) reduce the number of printed documents in America. D) enable tax collectors to become wealthy. E) raise taxes to a higher level than in Britain. Ans: A

334. Passage of the Sugar Act and the Stamp Act A) led many colonists to believe that the British were expanding colonial freedom. B) convinced many colonists that the British were trying to take away their historic liberty. C) resulted in fewer laws being passed by Parliament regarding the colonies. D) exemplified to many colonists the difference between legislation and taxation. E) required action by each colonial legislature. Ans: B

335. Unlike the _______________ Act, the _______________ Act and the _______________ Act were both indirect taxes on trade goods arriving in American ports. A) Townshend, Stamp, Sugar B) Stamp, Sugar, Townshend C) Stamp, Quartering, Townshend D) Declaratory, Stamp, Sugar E) Quartering, Stamp, Sugar Ans: B

336. Arrange the following events in chronological order: (A) Sugar Act, (B) Declaratory Act, (C) Stamp Act, (D) repeal of the Stamp Act. A) A, C, D, B B) C, A, D, B C) C, B, A, D D) B, A, C, D E) A, B, D, C Ans: A

337. Colonists objected to the Stamp Act because A) it was a very expensive tax. B) they believed it could not be repealed. C) Parliament passed the tax, not the colonists. D) they opposed all taxes. E) they wanted their independence. Ans: C

338. When colonists shouted No taxation without representation, they were rejecting Parliament's power to A) legislate for the colonies in any matter whatsoever. B) levy revenue-raising taxes on the colonies. C) enforce the old Navigation Laws. D) regulate trade in the empire. E) choose colonial legislators who would pass taxes. Ans: B

339. Actions taken by the colonists that helped them unite include A) the Stamp Act Congress. B) nonimportation agreements. C) spinning bees. D) the making and wearing of homemade woolen goods. E) all of the above. Ans: E

340. Virtual representation meant that A) almost all British subjects were represented in Parliament. B) every member of Parliament represented all British subjects. C) colonists could elect their own representatives to Parliament. D) Parliament could pass virtually all types of legislation except taxes. E) each member of Parliament represented only people in his district. Ans: B

341. Colonial protest against the Stamp Act took the form of A) convening a colonial congress to request repeal of the act. B) a colonial boycott against British goods. C) violence in several colonial towns. D) wearing homemade woolen clothes. E) all of the above. Ans: E

342. As a result of American opposition to the Townshend Acts, A) British officials sent regiments of troops to Boston to restore law and order. B) the port of Boston was closed. C) Americans killed several British soldiers in the Boston Massacre. D) Parliament repealed all of the taxes levied under this legislation. E) Prime Minister Townshend was forced to resign. Ans: A

343. The colonists took the Townshend Acts less seriously than the Stamp Act because A) they saw the futility of resistance. B) smuggling was nearly impossible. C) it was a direct tax. D) the items taxed were rarely used. E) it was light and indirect. Ans: E

344. Arrange these events in chronological order: (A) Boston Massacre, (B) Townshend Acts, (C) Tea Act, (D) Intolerable Acts. A) A, B, C, D B) D, B, C, A C) C, B, D, A D) B, A, C, D E) A, C, D, B Ans: D

345. Match each individual on the left with the correct description.

A. Samuel Adams B. John Adams C. Crispus Attucks A) A-4, B-3, C-2 B) A-3, B-4, C-1 C) A-2, B-4, C-2 D) A-2, B-1, C-3 E) A-4, B-1, C-2 Ans: B

1. a casualty of the Boston Massacre 2. a foreign volunteer who drilled American troops during the War of Independence 3. a pamphleteer who first organized committees to exchange ideas and information on resisting British policy 4. a Massachusetts politician who opposed the moderates' solution to the imperial crisis at the First Continental Congress

346. The tax on tea was retained when the Townshend Acts were repealed because A) Parliament believed the colonists would not object. B) the money was needed to support troops. C) it kept alive the principle of parliamentary taxation. D) it was the only tax passed by the colonists. E) colonial governors requested it. Ans: C

347. The local committees of correspondence organized by Samuel Adams A) promoted his bid to become governor of Massachusetts. B) promoted independent action in each colony to support the British. C) kept opposition to the British alive, through exchange of propaganda. D) served as a precursor to the United States Postal Service. E) led the Boston Massacre. Ans: C

348. Arrange the following events in chronological order: (A) clash at Lexington and Concord, (B) meeting of the First Continental Congress, (C) Quebec Act, (D) Boston Tea Party. A) C, D, A, B B) B, A, C, D C) D, C, B, A D) A, B, D, C E) A, D, C, B Ans: C

349. When Parliament passed the Tea Act, colonists A) rejoiced that Parliament had seemingly accepted the American definition of representation. B) suspected that it was a trick to get them to violate their principle of No taxation without representation. C) immediately called the First Continental Congress into session. D) avoided the tax on tea by buying their tea directly from the British East India Company. E) gave up tea and turned to coffee. Ans: B

350. The Boston Tea Party of 1773 was A) an isolated incident. B) directed only at the British East India Company. C) not the only such protest to occur. D) supported by friends of America in Britain. E) the result of the Intolerable Acts. Ans: C

351. The most drastic measure of the Intolerable Acts was the A) Quartering Act. B) Quebec Act. C) Sugar Act. D) Courts Act. E) Boston Port Act. Ans: E

352. The Quebec Act A) outlawed Catholicism in British Quebec. B) denied Quebec a representative assembly and trial by jury. C) restricted Quebec's boundaries to the area north of the Great Lakes. D) was generally ignored by the thirteen seaboard colonies because it had little effect on their relations with Britain. E) granted Quebec a representative assembly and trial by jury. Ans: B

353. The Quebec Act was especially unpopular in the American colonies because it did all of the following except A) turn an extensive amount of territory over to Catholic control. B) affect many colonies, not just Massachusetts. C) deny the French the right to retain many of their old customs. D) alarm land speculators, who saw a huge area snatched from their grasp. E) it set a dangerous precedent against jury trials. Ans: C

354. The First Continental Congress was called in order to A) consider ways of redressing colonial grievances. B) become a legislative body. C) write the Declaration of Independence. D) decide which of Parliament's taxes the colonies would and would not pay. E) help implement provisions of the Quebec Act. Ans: A

355. The First Continental Congress A) was attended by delegates from each of the thirteen colonies. B) adopted a moderate proposal for establishing a kind of home rule for the colonies under British direction. C) made a ringing declaration of America's independence from Britain. D) called for a complete boycott of British goods. E) adjourned shortly after convening. Ans: D

356. As a result of Parliament's rejection of the petitions of the Continental Congress, A) Americans reluctantly obeyed the British laws. B) fighting and bloodshed took place, and war began. C) Sam Adams and John Hancock were arrested. D) America sent new petitions to Parliament. E) Ben Franklin returned to the colonies since his efforts failed. Ans: B

357. As the War for Independence began, Britain had the advantage of A) overwhelming national wealth and naval power. B) an alliance with Spain and Holland. C) a well-organized and united home government and population. D) first-rate generals and a well-supplied professional army. E) all of the above. Ans: A

358. All of the following were weaknesses of the British military during the War for Independence except A) second-rate officers. B) soldiers who were incapable of fighting effectively. C) the need to keep many soldiers in Europe in case of trouble. D) the long supply lines. E) brutal treatment of their soldiers. Ans: B

359. Many Whigs in Britain hoped for an American victory in the War for Independence because they A) favored French domination of North America. B) were strongly pacifist. C) feared that if George III triumphed, his rule at home might become tyrannical. D) rejected colonialism. E) opposed the mercantilist system. Ans: C

360. As the War for Independence began, the colonies had the advantage of A) highly reliable and well-supplied troops. B) potential aid from the Armed Neutrality League. C) a well-organized, strongly committed, and united population. D) many outstanding civil and military leaders. E) able naval leaders. Ans: D

361. The colonists faced all of the following weaknesses in the War for Independence except A) poor organization. B) sectional jealousy, which constantly interfered with the appointment of military leaders. C) great difficulties in raising money to support the army. D) the use of numerous European officers. E) a weak central authority running the war effort. Ans: D

362. By the end of the War for Independence, A) the majority of Americans supported independence with selfless devotion. B) America had an army larger than Britain's. C) the American military no longer needed foreign assistance. D) a few thousand American regular troops were finally whipped into shape. E) America had built a strong navy. Ans: D

363. African Americans during the Revolutionary War A) fought for both the Americans and the British. B) fought only for the British. C) fought only for the Americans. D) supported neither side, as both enslaved them. E) seized the opportunity to gain their freedom by running away to Barbados. Ans: A

364. Regarding American independence, A) a majority of Americans supported the cause selflessly. B) most of the American business community sacrificed profit for victory. C) France gave little assistance. D) only a select minority supported independence with selfless devotion. E) Spain was in total opposition. Ans: D

365. When the Second Continental Congress met in 1775, A) its members felt a strong desire for independence. B) it cut off communications with the British government. C) it continued to stall on the creation of an army and navy. D) there was no well-defined sentiment for independence. E) the conservative element was weakened. Ans: D

366. Perhaps the most important single action of the Second Continental Congress was to A) select George Washington to head the army. B) draft new appeals to the king. C) adopt measures to raise money. D) postpone an immediate demand for independence. E) support independence. Ans: A

367. Arrange the following events in chronological order: (A) fighting at Lexington and Concord, (B) convening of the Second Continental Congress, (C) publication of Common Sense, (D) adoption of the Declaration of Independence. A) B, C, A, D B) A, B, C, D C) A, C, D, B D) C, D, A, B E) A, B, D, C Ans: B

368. As commander of America's Revolutionary army, George Washington exhibited all of the following except A) military genius. B) courage. C) a sense of justice. D) moral force. E) patience. Ans: A

369. The Revolutionary War began with fighting in __________; then in 17771778, fighting was concentrated in __________; and the fighting concluded in __________. A) the South, the middle colonies, New England B) the middle colonies, New England, the South C) New England, the South, the middle colonies D) New England, the middle colonies, the South E) the middle colonies, the South, New England Ans: D

370. George Washington's selection to lead the colonial army was A) a poor choice. B) largely political. C) based solely on military experience. D) opposed by New Englanders. E) done with no misgivings. Ans: B

371. In 1775, once fighting between the colonies and Great Britain began, A) America immediately declared its independence. B) the tempo of warfare diminished. C) the colonists denounced the Parliament. D) the colonists affirmed their loyalty to the King. E) all of the above. Ans: D

372. The colonial army eventually lost the Battle of Bunker Hill because its troops were A) outnumbered. B) short of gunpowder. C) poorly organized. D) poor shots. E) lacking in courage. Ans: B

373. King George III officially declared the colonies in rebellion just after A) the armed clash at Lexington and Concord. B) the First Continental Congress convened. C) the Battle of Bunker Hill. D) Benedict Arnold's forces' captured Ticonderoga and Crown Point. E) hiring Hessian solders to fight in America. Ans: C

374. The Olive Branch Petition A) was passed by Parliament. B) was an expression of King George III's desire for peace. C) promised no treason charges if colonists stopped fighting. D) was an attempt by the colonists to gain support of Native Americans. E) professed American loyalty to the crown. Ans: E

375. With the American invasion of Canada in 1775, A) the French Canadians took the opportunity to revolt against British control. B) Benedict Arnold seized the occasion to desert to the British. C) the colonials' claim that they were merely fighting defensively for a redress of grievances was contradicted. D) the Revolution became a world war. E) George III declared the colonies in rebellion. Ans: C

376. The colonists' invasion of Canada in 1775 A) contradicted the American claim that they were only fighting defensively. B) was of little strategic value for the colonists. C) was eagerly welcomed by French-Canadian leaders. D) resulted in the capture of both Montreal and Quebec. E) resulted in Benedict Arnold's defection to Great Britain. Ans: A

377. The colonists delayed declaring their independence until July 4, 1776, for all of the following reasons except A) lack of military victories. B) support for the tradition of loyalty to the empire. C) the realization that the colonies were not united. D) fear of British military reprisals. E) a continued belief that America was part of the transatlantic community. Ans: A

378. One purpose of the Declaration of Independence was to A) warn other nations to stay out of the Revolution. B) ask for an end to slavery. C) appeal for fairer treatment by Parliament. D) explain to the rest of the world why the colonies had revolted. E) condemn Parliament for its actions. Ans: D

379. In a republic, power A) comes from the aristocrats. B) comes from a select few based on religion. C) comes from the people themselves. D) resides in property owners. E) belongs only to the educated. Ans: C

380. Thomas Paine's pamphlet Common Sense A) was published before any fighting took place between the colonists and the British. B) remained unpopular for several years before being accepted by the public. C) called for American independence and the creation of a democratic republic. D) called on the British people to overthrow the king. E) led to Paine's eventual arrest and imprisonment in America. Ans: C

381. Thomas Paine argued that all government officials A) were corrupt. B) should derive their authority from popular consent. C) should be part of a natural aristocracy. D) need not listen to the voice of the uneducated. E) should not be paid for their service. Ans: B

382. The resolution that These United Colonies are, and of right ought to be, free and independent states was introduced into the Second Continental Congress by Virginia delegate A) Patrick Henry. B) Thomas Jefferson. C) Richard Henry Lee. D) Thomas Paine. E) John Adams. Ans: C

383. The feasibility of representative government had been demonstrated in the A) militia movement. B) Olive Branch Petition. C) Declaration of Independence. D) committees of correspondence. E) all of the above. Ans: D

384. Examples of colonial experience with selfgovernance, which prepared Americans for a republic, included all of the following except A) New England town meetings. B) committees of correspondence. C) militia service. D) the relative equality of landowning farmers. E) the absence of a hereditary aristocracy. Ans: C

385. Most Americans considered which of the following to be fundamental for any successful republican government? A) a wealthy class to govern B) the primacy of the property rights of individuals C) primacy of the interests of individuals D) retention of a constitutional monarchy E) civic virtue Ans: E

386. When America became a republic and political power no longer rested with an all-powerful king, A) the American colonies were able to gain their independence. B) England experienced the Glorious Revolution. C) individuals needed to sacrifice their own self-interest to the public good. D) chaos gripped the nation. E) all of the above. Ans: C

387. Which individual privately advocated equality for women? A) Betsy Ross B) Thomas Jefferson C) Martha Washington D) Benjamin Franklin E) Abigail Adams Ans: E

388. The Declaration of Independence did all of the following except A) invoke the natural rights of humankind to justify revolt. B) catalog the tyrannical actions of King George III. C) argue that royal tyranny justified revolt. D) offered the British one last chance at reconciliation. E) accused the British of violating the natural rights of the Americans. Ans: D

389. Americans who opposed independence for the colonies were labeled __________ or _______________, and the independenceseeking Patriots were also known as _______________. A) Tories, Whigs, Loyalists B) Loyalists, Tories, Whigs C) Whigs, Tories, Loyalists D) Loyalists, Whigs, Tories E) Sons of Liberty, Tories, Whigs Ans: B

390. Like many revolutions, the American Revolution was A) a majority movement. B) a minority movement. C) started by forces outside the country. D) one in which little attention was given to those civilians who remained neutral. E) one which produced a minimum of violence. Ans: B

391. The Patriot militia played a crucial role in the Revolution in all of the following ways except A) taking up the task of political education. B) raising funds to support the war effort. C) convincing people that the British army was an unreliable friend. D) mercilessly harassing small British detachments. E) as effective agents of Revolutionary ideas. Ans: B

392. The Americans who continued to support the crown after independence had been declared were more likely to be all of the following except A) well educated. B) from among the older generation. C) affiliated with the Anglican church. D) from New England. E) wealthy. Ans: D

393. Many Americans remained loyalists during the Revolution for all of the following reasons except A) fear of retribution. B) they believed a Patriot victory would lead to anarchy. C) some were promised freedom. D) they believed the British would preserve religious toleration. E) they believed in British military superiority. Ans: A

394. Which of the following fates befell Loyalists after the Revolutionary War? A) Some fled to England. B) Some re-established themselves in America. C) Some had their property confiscated. D) Some were exiled. E) all of the above. Ans: E

395. All of the following fates befell colonial Loyalists except A) tarring and feathering. B) subjection to a reign of terror. C) imprisonment. D) exile. E) riding astride fence rails. Ans: B

396. Loyalists were least numerous in A) New York B) Pennsylvania. C) Virginia. D) the middle colonies. E) New England. Ans: E

397. To help the British, colonial Loyalists did all of the following except A) fight for the British. B) serve as spies. C) pay extra taxes to fund the war. D) keep Patriot soldiers at home to protect their families. E) incite the Indians. Ans: C

398. General William Howe did not pursue and defeat George Washington's army after the Battle of Long Island for all of the following reasons except A) he remembered the slaughter of Bunker Hill. B) the country was rough. C) supplies were slow in coming. D) he did not relish the rigors of a winter campaign. E) he lacked sufficient naval support. Ans: E

399. In late 1776 and early 1777, George Washington helped restore confidence in America's military by A) defeating the Hessians at Trenton and the British at Princeton. B) securing the support of France for the American war effort with a victory in New York City. C) gaining a pay raise for American troops. D) bringing in Alexander Hamilton as his aide. E) providing adequate food and clothing for the soldiers. Ans: A

400. The basic strategy of the British in 1777 was to try to A) control the Delaware Valley. B) invade the southern colonies. C) isolate New England. D) hold the cities and let colonists control the countryside. E) isolate the South. Ans: C

401. Match each British general below with the battle in which he was involved. A. William Howe1. SaratogaB. John Burgoyne2. YorktownC. Charles Cornwallis3. Long IslandD. Nathanael Greene A) A-1, B-2, C-3 B) A-3, B-1, C-2 C) A-3, C-2, D-1 D) B-1, C-2, D-3 E) C-1, B-2, D-3 Ans: B

402. Arrange these battles in chronological order: (A) Trenton, (B) Saratoga, (C) Long Island, (D) Charleston. A) B, C, A, D B) C, A, B, D C) C, B, A, D D) C, B, D, A E) A, B, C, D Ans: B

403. The basic principles of the Model Treaty and the new philosophy behind American international affairs, contained all of the following except A) no political connection. B) no military connection. C) only commercial connection. D) no economic connection. E) novus ordo seculorum a new order for the ages. Ans: D

404. The Battle of Saratoga was a key victory for the Americans because it A) brought the British to offer recognition of colonial independence. B) brought the colonists much-needed aid and a formal alliance with France. C) prevented the fighting from spreading into the southern colonies. D) prevented the colonial capital from being captured by the British. E) kept Benedict Arnold from joining the British. Ans: B

405. The basic principles in the Model Treaty A) were considered old-fashioned and out-dated. B) were self-denying restrictions to the Americans. C) were not popular among most enlightened figures in America. D) held that military conflict would still determine international relations among countries. E) infused an element of realism into American attitudes toward international affairs that proved short-sighted and inconsistent. Ans: B

406. France came to America's aid in the Revolution because A) French officials supported our cause of democracy. B) it hoped to gain access to the American fur trade. C) it wanted revenge against the British. D) it could use America to test new military tactics. E) all of the above. Ans: C

407. America's first entangling alliance was with A) Great Britain. B) France. C) Spain. D) Holland. E) Russia. Ans: B

408. Who was the American diplomat that negotiated the Model Treaty with France? A) John Adams B) Thomas Jefferson C) Thomas Paine D) Benjamin Franklin E) Patrick Henry Ans: D

409. The Armed Neutrality League was started by A) Louis XIV of France. B) Charles V of Spain. C) Catherine the Great of Russia. D) King Leopold of Belgium. E) George III of Britain. Ans: C

410. When the alliance with France was formalized, the Americans were able to A) gain access to large sums of money. B) double the size of their fighting forces. C) avail themselves of French naval strength. D) gain immense amounts of equipment. E) all of the above. Ans: E

411. The commander of French troops in America was A) Rochambeau. B) Lafayette. C) de Grasse. D) Burgoyne. E) Howe. Ans: A

412. French aid to the colonies A) greatly aided America's struggle for independence. B) was motivated by what the French considered to be their own national interests. C) forced the British to change their military strategy in America. D) helped them protect their own West Indies islands. E) all of the above. Ans: E

413. Shortly after French troops arrived in America, the resulting improvement in morale staggered when A) America discovered the true reasons motivating France's assistance. B) General Benedict Arnold turned traitor. C) General Nathanael Greene lost Georgia to the British. D) the French began to win battles that the Americans had been unable to win. E) the Armed Neutrality League sided with Britain. Ans: B

414. The colonists suffered their heaviest losses of the Revolutionary War at the Battle of A) Charleston. B) Cowpens. C) Valley Forge. D) Long Island. E) Brandywine Creek. Ans: A

415. Match each individual below with the correct descriptive phrase. A. George Rogers Clark B. Nathanael Greene C. John Paul Jones 1. commanded the Patriot invasion of Canada 2. commanded Patriot troops in the South 3. commanded Patriot troops in the West 4. commanded Patriot naval forces A) A-4, B-3, C-l B) A-2, B-1, C-4 C) A-3, B-2, C-4 D) A-1, B-4, C-3 E) A-4, B-3, C-2 Ans: C

416. Some Indian nations joined the British during the Revolutionary War because A) the British threatened them with destruction if they did not help. B) they believed that a British victory would restrain American expansion into the West. C) the British hired them as mercenaries. D) they were bound by treaties. E) none of the above. Ans: B

417. The Fighting Quaker who cleared most of Georgia and South Carolina was A) Charles Cornwallis. B) Benedict Arnold. C) Joseph Brant. D) Benjamin Smith. E) Nathanael Greene. Ans: E

418. The Indian chief who fought for the British in New York and Pennsylvania was A) Seneca. B) Pontiac. C) Joseph Brant. D) King Philip. E) Cowpens. Ans: C

419. The Treaty of Fort Stanwix, the first treaty between the United States and an Indian nation, resulted in A) the ceding of most of the Iroquois' land. B) an end to the practice of scalping. C) the slowing of the westward movement of pioneers. D) the renunciation by the Oneidas and the Tuscaroras of their support for the British. E) turning over the hair buyers for prosecution. Ans: A

420. During the Revolution, the frontier saw much fighting, which A) slowed the westward advance of the pioneers. B) caused most of the Indians to join the colonists' cause against the British. C) led to George Rogers Clark's downfall as a military leader. D) failed to stem the tide of westward-moving pioneers. E) ultimately led Benedict Arnold to go over to the British. Ans: D

421. The most important contribution of the seagoing privateers during the Revolutionary War was that they A) gained control of the sea for the colonists. B) successfully invaded the British West Indies. C) captured hundreds of British merchant ships. D) fought the British navy to a standstill. E) made reliance on the French unnecessary. Ans: C

422. After the British defeat at Yorktown, A) the fighting continued for more than a year. B) the war ended within a month. C) the French withdrew their assistance as it was no longer needed. D) King George III decided to end the struggle. E) Spain finally entered the war on our side. Ans: A

423. American diplomats to the peace negotiations in Paris in 1782-1783 were instructed by the Second Continental Congress to A) accept any British offer that would essentially return British-American relations to their pre-1763 status. B) demand British cession of the trans-Allegheny West to the colonies. C) get the colonies out of their obligations under the Franco-American alliances. D) consult with the colonies' French allies and make no separate peace arrangements with the British. E) follow the lead of Spain, not France. Ans: D

424. Britain gave America generous terms in the Treaty of Paris because British leaders A) realized that they had been beaten badly. B) wanted to help Spain as well. C) had changed from Whig to Tory. D) were trying to persuade America to abandon its alliance with France. E) feared continued war might lead to a loss of their Latin American colonies. Ans: D

425. Regarding the provisions of the 1783 Treaty of Paris, which formally ended the Revolution, A) America faithfully adhered to each one. B) France was pleased with the results. C) America broke the assurances regarding treatment of the Loyalists. D) Spain gained all it wanted. E) America followed French instructions to the letter. Ans: C

426. Continental army officers attempting to form the Society of the Cincinnati A) were brought to trial for trying to sabotage the civil government. B) were ridiculed for their lordly pretensions. C) were trying to force the Congress to pay them their pensions. D) reflected the Revolutionary War generation's spirit of equality. E) represented the best of the officer corps. Ans: B

427. The American Revolution was A) truly radical. B) inconsequential in world history. C) an example of accelerated evolution rather than outright revolution. D) very much like the French revolution. E) very much like the Russian revolution. Ans: C

428. The world's first antislavery society was founded by A) Thomas Jefferson. B) Quakers in Philadelphia. C) Puritans in New England. D) Catholics in Maryland. E) the Congregational church. Ans: B

429. As part of the egalitarian movement of the American Revolution, A) several northern states abolished slavery. B) most states outlawed the overseas trade in indentured servants. C) many states repealed laws against interracial marriage. D) some southern states passed legislation providing for the gradual abolition of slavery. E) laws against interracial marriage were eliminated. Ans: A

430. Early signs of the abolitionist movement can be seen in the A) Articles of Confederation. B) Constitution. C) emancipation of some slaves. D) passage of laws allowing interracial marriage. E) abolition of slavery in a few southern states. Ans: C

431. The Founding Fathers failed to eliminate slavery because A) they did not truly believe in democracy. B) a fight over slavery might destroy national unity. C) they were more concerned with securing equality for women. D) the North, as its industry expanded, began to rely more heavily on slave labor. E) economic conditions would not allow such a loss. Ans: B

432. The struggle for divorce between religion and government proved fiercest in A) Georgia B) Virginia C) Pennsylvania D) New York E) Maryland Ans: B

433. As a result of the Revolution's emphasis on equality, all of the following were achieved except A) the reduction of property qualifications for voting by most states. B) the growth of trade organizations for artisans and laborers. C) the establishment of the world's first antislavery society. D) full equality between women and men. E) abolishing medieval inheritance laws. Ans: D

434. The most important outcome of the Revolution for white women was that they A) permanently gained the right to vote. B) were allowed to serve in the national legislature. C) were elevated as special keepers of the nation's conscience. D) finally gained fully equal status with white males. E) were given the right to vote in some states. Ans: C

435. As written documents, the state constitutions were intended to A) represent a fundamental law superior to ordinary legislation. B) be subordinate to state laws. C) grant the governor more power than the legislature. D) keep the government in the hands of the well-to-do. E) reaffirm states' rights. Ans: A

436. As a means of ensuring that legislators stay in touch with the mood of the people, state constitutions A) were rewritten once every ten years. B) were rewritten once every five years. C) required yearly visits to the homes of their constituents. D) stipulated that ordinary legislation could override the constitution. E) required the annual election of legislators. Ans: E

437. As a result of the Revolution, many state capitals were relocated westward A) because better roads now made this territory more easily accessible. B) due to a fear of British capture. C) because water routes were now opened to the interior regions D) to get them away from the haughty eastern seaports. E) all of the above. Ans: D

438. One reason that the United States avoided the frightful excesses of the French Revolution is that A) America declared martial law until the Constitution was enacted in 1789. B) the American Revolution suddenly overturned the entire political framework. C) cheap land was easily available. D) political democracy preceded economic democracy. E) a strong sense of class consciousness already existed. Ans: C

439. It was highly significant to the course of future events that A) political democracy preceded economic democracy in the United States. B) deflation rather than inflation resulted from the Revolution. C) no economic depression occurred as a consequence of the Revolution. D) economic democracy preceded political democracy in the United States. E) the United States went off the gold standard after the Revolution. Ans: D

440. The economic status of the average American at the end of the Revolutionary War was A) better than before the war. B) probably worse than before the war. C) about the same as before the war. D) more closely tied to Britain than before the war. E) more closely tied to France than before the war. Ans: B

441. Immediately after the Revolution, the new American nation's greatest strength lay in its A) ingrained respect for authority. B) excellent political leadership. C) lack of inhibiting political heritage. D) sound economic structure. E) economic ties to France. Ans: B

442. The Second Continental Congress of Revolutionary days A) operated with strong constitutional authority. B) still did not comprise representatives from all thirteen states. C) took away the sovereignty of the states. D) was little more than a conference of ambassadors with very limited power. E) did little of lasting value. Ans: D

443. The Articles of Confederation were finally approved when A) agreement was reached on who would be president. B) states gave up their right to coin money. C) all states claiming western lands surrendered them to the national government. D) the states gave up their power to establish tariffs. E) a two-house national legislature was added. Ans: C

444. The major issue that delayed ratification of the Articles of Confederation concerned A) taxation. B) tariff policy. C) monetary policy. D) western lands. E) monetary standards. Ans: D

445. The Articles of Confederation left Congress unable to A) organize development of the western lands. B) deal with foreign affairs. C) apportion state representation equally. D) enforce a tax-collection program. E) establish a postal service. Ans: D

446. A major strength of the Articles of Confederation was its A) control over interstate commerce. B) strong judicial branch. C) presentation of the ideal of a united nation. D) ability to coin money. E) strong executive branch. Ans: C

447. The Northwest Ordinance of 1787 A) provided for the survey and sale of public lands in the Old Northwest. B) established a procedure for governing the Old Northwest territory. C) banned slavery from all territories of the United States. D) cleared the way for ratification of the Articles of Confederation. E) gave control over land to the territories in which they were located. Ans: B

448. One of the most farsighted provisions of the Northwest Ordinance of 1787 A) set aside a section of each township for education. B) abolished slavery in all of the United States. C) prohibited slavery in the Old Northwest. D) kept power in the national government. E) none of the above. Ans: C

449. The Land Ordinance of 1785 provided for all of the following except A) money from land sales should be used to pay off the national debt. B) the land should be surveyed before its sale. C) the territory should be divided into townships six miles square. D) the sixteenth section should be sold to support education. E) prohibiting slavery. Ans: E

450. Match each nation on the left with the correct description of the problem it presented for U.S. foreign relations following the Revolutionary War. A. Britain B. France C. Spain D. Barbary Coast 1. threatened American commerce in the Mediterranean 2. demanded repayment of wartime loans 3. occupied a chain of trading forts in the Old Northwest 4. controlled important trade routes from the interior of North America A) A-1, B-3, C-2, D-4 B) A-2, B-4, C-1, D-3 C) A-2, B-2, C-3, D-4 D) A-3, B-2, C-4, D-1 E) A-4, B-2, C-1, D-3 Ans: D

451. After the Revolutionary War, both Britain and Spain A) tried to gain control of Florida. B) did their best to win the friendship of America. C) prevented America from exercising effective control over about half of its total territory. D) helped America to fight the pirates in North America. E) abandoned their fortifications in the Old Northwest. Ans: C

452. Shays's Rebellion was provoked by A) fear that the Articles of Confederation had created too strong a national government for the United States. B) efforts by wealthy merchants to replace the Articles of Confederation with a new constitution. C) a quarrel over the boundary between Massachusetts and Vermont. D) foreclosures on the mortgages of backcountry farmers. E) the government's failure to pay bonuses to Revolutionary War veterans. Ans: D

453. Shays's Rebellion convinced many Americans of the need for A) lower taxes. B) granting long-delayed bonuses to Revolutionary War veterans. C) a vigilante effort by westerners to halt the Indian threat. D) a stronger central government. E) a weaker military presence in the West. Ans: D

454. Under the Articles of Confederation, the relationship between the thirteen states A) improved to the point of total unity. B) was good economically but poor politically. C) led to a single currency. D) convinced many that a stronger central government was needed. E) was good politically but poor economically. Ans: D

455. The debate between the supporters and critics of the Articles of Confederation centered on how to A) reconcile states' rights with strong national government. B) transfer territories to equal statehood. C) abolish slavery yet preserve national unity. D) balance the power of legislative and executive offices of government. E) conduct foreign policy while remaining neutral. Ans: A

456. The issue that finally touched off the movement toward the Constitutional Convention was A) control of public lands. B) control of commerce. C) Indian policy. D) monetary policy. E) foreign threats to our independence. Ans: B

457. By the time the Constitution was adopted in 1789, A) the American economy was continuing to experience problems. B) prosperity was beginning to return. C) foreign trade was still in terrible shape. D) inflation was continuing to increase. E) the issue of states' rights had all but disappeared. Ans: B

458. The Constitutional Convention was called to A) write a completely new constitution. B) allow the most radical Revolutionary leaders to write their ideas into law. C) weaken the power of the central government. D) revise the Articles of Confederation. E) reassess our foreign alliances. Ans: D

459. Which of the following Revolutionary leaders was NOT present at the Constitutional Convention? A) Thomas Jefferson B) Benjamin Franklin C) James Madison D) George Washington E) Alexander Hamilton Ans: A

460. The delegate whose contributions to the Philadelphia Convention were so notable that he has been called the Father of the Constitution was A) George Washington. B) Benjamin Franklin. C) James Madison. D) Thomas Jefferson. E) Patrick Henry. Ans: C

461. The delegates at the Constitutional Convention were concerned mainly with A) abolishing slavery. B) establishing a very powerful military. C) protecting America from its weaknesses abroad and its excesses at home. D) ensuring that the states continue to control tariff policies. E) establishing the principle of states' rights. Ans: C

462. Most of the delegates at the Constitutional Convention could best be labeled A) states' rightists. B) antifederalists. C) nationalists. D) ordinary citizens. E) counter revolutionaries. Ans: C

463. Motives of the delegates to the 1787 Constitutional Convention in Philadelphia include all of the following except A) to preserve the union. B) to forestall anarchy. C) to ensure the security of life and property. D) to curb unrestrained democracy E) to increase individual freedom. Ans: E

464. The large-state plan put forward in the Constitutional Convention A) ultimately provided the framework of the Constitution. B) was proposed by Patrick Henry. C) favored states such as New Jersey. D) favored southern states over northern states. E) based representation in the House and Senate on population. Ans: E

465. The Great Compromise at the Constitutional Convention worked out an acceptable scheme for A) regulating interstate commerce. B) levying taxes for raising a militia. C) apportioning congressional representation. D) electing the president. E) choosing Senators. Ans: C

466. Under the Constitution, the president of the United States was to be elected by a majority vote of the A) general public. B) Senate. C) Electoral College. D) House of Representatives. E) state legislatures. Ans: C

467. The idea that all tax measures should start in the House was made to appease A) the least populated states. B) western states. C) eastern states. D) the industrialists. E) the big states with the most people. Ans: E

468. The Constitutional Convention addressed the North-South controversy over slavery through the A) large-state plan. B) small-state plan. C) three-fifths compromise. D) closing of the slave trade until 1807. E) Northwest Ordinance. Ans: C

469. Which of the following is a compromise in the Constitution? A) counting all slaves in apportioning membership in the House B) continuation of the foreign slave trade C) direct election of the president D) control of interstate commerce by the national government E) prohibiting states from abolishing slave trade Ans: B

470. By their actions, the delegates to the Constitutional Convention manifested their common beliefs in all of the following except A) government by the consent of the governed. B) checks and balances in government. C) manhood-suffrage democracy. D) the sanctity of private property. E) a stronger central government. Ans: C

471. The one branch of the government elected directly by the people is the A) military. B) House of Representatives. C) executive. D) judiciary. E) Senate. Ans: B

472. The new Constitution established the idea that the only legitimate government was one based on A) a strong central government. B) an unwritten constitution. C) the authority of the state. D) control by wealthier people. E) the consent of the governed. Ans: E

473. The ultimate guarantor of liberty and justice was A) the authority of the state. B) a written constitution. C) an independent judicial system. D) the virtue of the people. E) all of the above. Ans: D

474. The delegates at the Constitutional Convention stipulated that the new Constitution be ratified by A) state conventions. B) state legislatures. C) popular referendum. D) majority vote in the Congress. E) the judiciary. Ans: A

475. The antifederalist camp included all of the following groups except A) supporters of a strong central authority. B) states' rights supporters. C) backcountry dwellers. D) paper money advocates. E) debtors. Ans: A

476. Probably the most alarming characteristic of the new Constitution to those who opposed it was the A) creation of a federal district for the national capital. B) creation of a standing army. C) absence of a bill of rights. D) omission of any reference to God. E) creation of the presidency. Ans: C

477. Among other views, The Federalist, written during the ratification debate, argued that it was A) impossible to safeguard the rights of states from the power of a strong central government. B) possible to extend a republican form of government over a large territory. C) inevitable that slavery would be abolished in the new republic. D) illegal to replace the Articles of Confederation with a new constitution. E) best to establish a direct democracy. Ans: B

478. Antifederalists believe that the sovereignty of the people resided in which branch of the central government? A) executive B) legislative C) judicial D) cabinet E) all of the above Ans: B

479. The federalists believe that the sovereignty of the people resided in which branch of the central government? A) executive B) legislative C) judicial D) none of the above. E) all of the above Ans: E

480. One of the enduring paradoxes of American history is that A) conservatives supported democracy. B) liberals supported democracy. C) both liberals and conservatives have championed the heritage of democratic revolution. D) conservatives and liberals were on opposite sides in the Revolution. E) conservatives opposed democracy. Ans: C

481. When the new government was launched in 1789, A) the nation's population was doubling about every twenty-five years. B) most people lived in the fast-growing cities. C) most people lived west of the Allegheny Mountains. D) New York was the largest city in the nation. E) Great Britain refused to establish diplomatic relations with the United States. Ans: A

482. Regarding central authority, early Americans saw it as all of the following except A) something to be ultimately eliminated. B) something to be distrusted. C) something to be watched. D) something to be curbed. E) a necessary evil. Ans: A

483. The new Constitution did not provide for the creation of a(n) A) Electoral College. B) vice president. C) Supreme Court. D) cabinet. E) federal court system. Ans: D

484. Despite the flourishing cities, America's population was still about __________ rural. A) 20% B) 40% C) 55% D) 70% E) 90% Ans: E

485. Match the individual with his office in the new government. A. Thomas Jefferson1. attorney generalB. Alexander Hamilton 2. secretary of stateC. Henry Knox3. secretary of war4. secretary of treasury A) A-1, B-3, C-2 B) A-3, B-1, C-4 C) A-2, B-4, C-3 D) A-4, B-2, C-l E) A-1, B-4, C-3 Ans: C

486. One of the major criticisms of the Constitution as drafted in Philadelphia was that it A) was too long and detailed. B) was far too short and required more detail. C) failed to guarantee property rights. D) failed to provide a mechanism for amendment. E) did not provide guarantees for individual rights. Ans: E

487. The Bill of Rights was intended to protect __________ against the potential tyranny of _________________________. A) the prerogatives of Congress, the president B) the army and the navy, the national government C) the South, the northern majority D) individual liberties, a strong central government E) civilian authorities, the military Ans: D

488. One of the first jobs facing the new government formed under the Constitution was to A) establish a powerful army. B) reestablish diplomatic ties with Britain. C) draw up and pass a bill of rights. D) establish economic ties with France. E) all of the above. Ans: C

489. All of the following are guarantees provided by the Bill of Rights except A) the right to vote for all citizens. B) freedom of speech. C) freedom of religion. D) freedom of the press. E) right to a trial by a jury. Ans: A

490. The__________ Amendment might rightly be called the states' rights amendment. A) First B) Sixth C) Ninth D) Tenth E) Eighth Ans: D

491. Alexander Hamilton's financial program for the economic development of the United States favored A) agricultural interests. B) trade with France. C) the wealthier class. D) the poor. E) the middle class. Ans: C

492. Hamilton believed that, together, his funding and assumption programs would A) gain the monetary and political support of the rich for the federal government. B) restore the principles of state sovereignty. C) be the quickest way to pay off the national debt. D) guarantee the fairest treatment of the original holders of government bonds. E) keep taxes low and therefore create a feeling of loyalty to the new federal government. Ans: A

493. As Secretary of the Treasury, Alexander Hamilton's first objective was to A) help the wealthy. B) bring more industry to the United States. C) see that more agricultural products were exported. D) bolster the national credit. E) put the country on the gold standard. Ans: D

494. All of the following were part of Alexander Hamilton's economic program except A) the creation of a national bank. B) funding the entire national debt at par. C) vigorous foreign trade. D) protective tariffs. E) paying only domestic debts but not foreign debts. Ans: E

495. Alexander Hamilton's financial plan for strengthening the economy and bolstering national credit proposed all of the following except A) funding the national debt. B) assuming state debts. C) abolishing tariffs. D) establishing a national bank. E) a low protective wall around infant industries. Ans: C

496. Alexander Hamilton believed that a limited national debt A) would do great harm to the nation's economy. B) might lead to military weakness. C) could persuade individuals and nations not to lend money to the United States. D) was beneficial, because people to whom the government owed money would work hard to make the nation a success. E) could help his economic plans but not his political plans. Ans: D

497. The aspect of Hamilton's financial program that received the least support in Congress was A) funding at par. B) assumption. C) the National Bank. D) a protective tariff. E) excise taxes. Ans: D

498. Hamilton expected that the revenue to pay the interest on the national debt would come from A) sales taxes and licensing fees. B) customs duties and excise tax. C) income and property taxes. D) western land sales and foreign loans. E) foreign aid. Ans: B

499. Alexander Hamilton's proposed bank of the United States was A) rejected by the House of Representatives. B) supported by Thomas Jefferson. C) enthusiastically supported by George Washington. D) based on the necessary and proper, or elastic, clause in the Constitution. E) never fully enacted. Ans: D

500. Which of the following pairs of items are not directly related to each other? A) implied powersnecessary and proper clause B) strict constructionTenth Amendment C) loose constructionelastic clause D) states' rightsloose construction E) necessary and proper clausevested powers Ans: D

501. Hamilton's major programs seriously infringed on A) checks and balances. B) national security. C) states' rights. D) free enterprise. E) federal authority. Ans: C

502. The Whiskey Rebellion of 1794 arose in southwestern Pennsylvania when the federal government A) levied an excise tax on whiskey. B) tried to prohibit the sale of whiskey. C) allowed the import of foreign whiskey. D) halted the export of American whiskey. E) tried to prohibit the manufacturing of whiskey. Ans: A

503. Alexander Hamilton's Bank of the United States was modeled on the A) Bank of England. B) Swiss National Bank. C) Bank of France. D) national bank that existed in the United States prior to the Constitution. E) National Bank of the Netherlands. Ans: A

504. The Founding Fathers had not envisioned the existence of permanent political parties because they A) opposed anyone who disagreed with them. B) were not part of the early colonial governments. C) had existed in Britain. D) saw opposition to the government as disloyal. E) all of the above. Ans: D

505. Match each political leader with his positions on public policy in the 1790s. A. Hamilton B. Jefferson 1. privileges for the upper classes 2. pro-British 3. sympathy for the common people 4. potent central government 5. pay off the national debt 6. government support for business 7. pro-French 8. universal education A) A-1, 2, 4, 6B-3, 5, 7, 8 B) A-1, 5, 6, 7B-2, 3, 4, 8 C) A-2, 3, 5, 8B-1, 4, 6, 7 D) A-3, 6, 7, 8B-1, 2, 4, 5 E) A-5, 2, 6, 3B-1, 4, 7, 8 Ans: A

506. Opposition by Thomas Jefferson and James Madison to the financial plan of Alexander Hamilton resulted in A) the formation of permanent political parties. B) Hamilton's dismissal from the cabinet by George Washington. C) politics drifting too far out of kilter with the wishes of the people. D) the rejection of Hamilton's plan by Washington. E) their dismissal from the cabinet of George Washington. Ans: A

507. The event of the 1790s that has left the deepest scar on American political and social life is A) the Whiskey Rebellion. B) the French Revolution. C) Hamilton's economic plan for the country. D) the trouble with Native Americans. E) the development of the political party system. Ans: B

508. The political party of the outs that provided the loyal opposition to the party in power in the 1790s was A) the Anti-Federalists. B) the Federalists. C) the Democratic-Republicans. D) the Whigs. E) the Tories. Ans: C

509. The Franco-American alliance of 1778 A) was ended by mutual agreement in 1789. B) bound the United States to neutrality in the event of war between France and Britain. C) bound the United States to help the French defend their West Indies. D) was invoked by the French to obtain American aid in France's war with Britain after 1793. E) led the United States to war with Great Britain in 1812. Ans: C

510. When the French Revolution developed into a war with Britain, George Washington and the American government A) supported Britain. B) assisted France militarily. C) tried to capture French possessions in North America and the West Indies. D) remained neutral. E) captured British possessions in North America. Ans: D

511. Washington's Neutrality Proclamation of 1793 A) was based on calculations of American self-interest. B) fulfilled America's obligations under the FrancoAmerican Treaty. C) was opposed by both Alexander Hamilton and Thomas Jefferson. D) dealt a severe blow to French military and naval strategists. E) had little impact on future American foreign policy. Ans: A

512. Arrange the following events in chronological order: (A) XYZ affair, (B) Neutrality Proclamation, (C) Jay's Treaty, (D) Kentucky and Virginia resolutions. A) C, B, A, D B) B, A, C, D C) B, C, A, D D) C, B, D, A E) A, B, D, C Ans: C

513. During its first quarter-century as a nation, one of the major problems facing America was A) the rivalry and warfare between France and Britain. B) a lack of good political leadership. C) the continued fighting between the United States and the Armed Neutrality League. D) Indian affairs. E) separation of church and state. Ans: A

514. Washington's Neutrality Proclamation clearly illustrated the truism that A) he was unprepared for the demands of foreign policy. B) foreign policy should be handled by a group and not by a single individual. C) the United States was trying to do what was best for its allies. D) self-interest is the basic cement of alliances. E) none of the above. Ans: D

515. The Treaty of Greenville signed in August with the Miami Confederation resulted in all of the following except A) giving to the United States vast tracts of land in the Old Northwest. B) the Indians receiving a $20,000 lump sum payment. C) an annual annuity of $9,000 to the Indians. D) the right of the Indians to hunt the land they had ceded. E) the establishment of an equal relationship with the Indians. Ans: E

516. Britain made neutrality very difficult for the United States during the French and British conflicts of the 1790s by A) granting America numerous trade privileges. B) seizing American merchant ships in the West Indies. C) leaving frontier outposts on American soil. D) helping to relieve tensions between Indians and Americans. E) blocking the major United States' seaports. Ans: B

517. Hamilton's position on the war between Britain and France in 1793 was primarily influenced by A) his commitment to the Franco-American alliance of 1778. B) the threat of British naval action against the American coast. C) the national government's dependence on customs collections for revenue. D) his personal commitment to democratic government as a world ideal. E) all of the above. Ans: C

518. In Jay's Treaty, the British A) pledged to stop seizing American ships. B) released Americans from their preRevolutionary War debt obligations to British merchants. C) promised to evacuate the chain of forts in the Old Northwest. D) refused to pay damages for seizures of American ships. E) were denied most favored nation status. Ans: C

519. The United States acquired free navigation of the Mississippi River in A) the Treaty of Greenville. B) Jay's Treaty. C) the Convention of 1800. D) the Pinckney Treaty. E) the Treaty of Paris. Ans: D

520. John Jay's 1794 treaty with Britain A) increased George Washington's huge popularity. B) provided further evidence of American support for France. C) alienated America from Spain. D) created deeper splits between Federalists and Democratic-Republicans. E) led to the election of Thomas Jefferson. Ans: D

521. One of George Washington's major contributions as president was A) keeping the nation out of foreign wars. B) the signing of Jay's Treaty. C) his advice against forming permanent alliances with foreign nations. D) securing a pledge from Britain to stop arming Indians on the western lands. E) establishing the political party system. Ans: A

522. Jay's Treaty contained all of the following provisions except A) a British promise to evacuate its chain of forts on U.S. soil. B) British consent to pay damages for the recent seizure of American ships. C) that Americans were bound to pay debts still owed to British merchants on pre-Revolutionary accounts. D) no promise by the British to pay for future seizure of American ships. E) a promise by the British to stop selling arms to the Indians. Ans: E

523. Washington's Farewell Address in 1796 A) warmly endorsed the appearance of two contending political parties in America. B) warned against the dangers of permanent foreign alliances. C) was delivered to a joint session of Congress by Washington himself. D) proposed a two-term limitation on the presidency. E) all of the above. Ans: B

524. In the election campaign of 1796, the DemocraticRepublicans made their primary issue A) the content of Washington's Farewell Address. B) Washington's refusal to consult Congress before issuing the Neutrality Proclamation. C) the terms of Jay's Treaty. D) the terms of the Pinckney Treaty. E) Alexander Hamilton's idea for a national bank. Ans: C

525. The 1796 presidential campaign focused heavily on A) the Bank of the United States. B) the candidates' personalities. C) slavery. D) foreign trade. E) real issues. Ans: B

526. The French grew angry with the United States after 1794 because A) of Jay's Treaty. B) Congress appointed second-rate ambassadors. C) of the XYZ affair. D) John Adams had been elected president. E) Thomas Jefferson was removed as ambassador. Ans: A

527. Foreign relations between the United States and France deteriorated in the late 1790s over A) the deportation of Citizen Gent. B) French seizure of American merchant ships. C) the adjustment of the Florida boundary. D) America's unilateral withdrawal from the Franco-American alliance. E) Pinckney's Treaty. Ans: B

528. The immediate cause of the undeclared war between the United States and France was A) the XYZ affair. B) the Gent mission. C) the Neutrality Proclamation. D) Washington's Farewell Address. E) Jay's Treaty. Ans: A

529. The United States finally negotiated a peace settlement with France in 1800 mainly because Napoleon A) had also reached a peace agreement with Britain. B) wanted to concentrate on gaining more power in Europe. C) realized that the French could not win a military victory over the American forces. D) had been convinced by the Democratic-Republican pleas for cooperation. E) had been removed from power. Ans: B

530. President Adams sought a peaceful solution to the undeclared war with France in order to A) ensure his chances of reelection in 1800. B) align himself with the Hamiltonian wing of the Federalist party. C) save the Franco-American alliance of 1778. D) prevent the outbreak of a full-scale war. E) keep trade with France in place. Ans: D

531. The main purpose of the Alien and Sedition Acts was to A) capture French and British spies. B) control the Federalists. C) silence and punish critics of the Federalists. D) keep Thomas Jefferson from becoming president. E) provide support for the DemocraticRepublican party. Ans: C

532. The Federalist-dominated Congress's Alien Act was aimed at ____________________, whereas the Sedition Act was primarily aimed at _____________________. A) rebellious slaves, newspapers B) recent immigrants, newspapers C) recent immigrants, merchants D) merchant smuggling, rebellious slaves E) Indians, farmers Ans: B

533. The Sedition Act A) threatened First Amendment freedoms. B) established criteria for deporting dangerous foreigners. C) changed naturalization requirements for new citizens. D) was never enforced. E) was found by the Supreme Court to be unconstitutional. Ans: A

534. The Virginia and Kentucky resolutions were written in response to A) the XYZ affair. B) Thomas Jefferson's presidential candidacy in 1800. C) the Alien and Sedition Acts. D) the compact theory of government. E) the Federalist papers. Ans: C

535. According to the compact theory advocated by Jefferson and Madison, A) the national government was the creation of the thirteen sovereign states. B) nullification was an invalid policy. C) the Virginia and Kentucky resolutions were illegal. D) legislation such as the Alien and Sedition Acts was proper. E) individuals, not the states, created the federal government. Ans: A

536. According to the Federalists, the duty of judging the unconstitutionality of legislation passed by Congress lay with A) state legislatures. B) the president. C) state supreme courts. D) the Supreme Court. E) the people. Ans: D

537. Federalist advocated rule by A) the majority. B) the best people. C) farmers. D) industrial workers. E) native born citizens only. Ans: B

538. Federalists strongly supported A) law and order. B) states' rights. C) strict construction. D) popular democracy. E) a weak military. Ans: A

539. For its continued success, Hamilton's financial program relied heavily on A) trade with Britain. B) removal of the Spanish from the Mississippi Valley. C) aid from France. D) retiring the national debt. E) high taxes. Ans: A

540. Hamiltonian Federalists advocated A) government interference in private enterprise. B) a strong central government. C) a full-blown democracy. D) strong ties with France. E) a low national debt. Ans: B

541. Thomas Jefferson appealed to all of the following groups except A) small shopkeepers. B) the underprivileged. C) the middle class. D) the upper class. E) artisans. Ans: D

542. To the Jeffersonian Republicans, the ideal citizen of a republic was a(n) A) seaboard merchant. B) town artisan. C) indentured servant. D) independent farmer. E) industrialist. Ans: D

543. Thomas Jefferson favored a political system in which A) the central government possessed the bulk of the power. B) cities were the primary focus of political activity. C) a large standing army ensured peace. D) the states retained the majority of political power. E) manufacturing interests dominated. Ans: D

544. Jeffersonians believed in all of the following except A) opposition to a national debt. B) agriculture as the ideal occupation. C) every adult white male's right to vote. D) freedom of speech. E) central authority should be kept to a minimum. Ans: C

545. Thomas Jefferson argued that a landless class of voters could be avoided in part by A) a redistribution of land. B) a reduced property tax. C) abolishing the property qualification to vote. D) continuing slavery. E) restricting the amount of property owned by each citizen. Ans: D

546. One of the first lessons learned by the Jeffersonians after their victory in the 1800 presidential election was A) the need to strengthen diplomatic ties with Britain. B) to go off the gold standard. C) to decrease tariffs. D) to institute an excise tax. E) that it is easier to condemn from the stump than to govern consistently. Ans: E

547. One of the greatest problems that John Adams and the Federalists faced in the election of 1800 was A) Adams's efforts to get America involved in a war with France. B) increased public debt brought on by war preparations. C) Adams's refusal to take the country to war against France. D) Alexander Hamilton's support of Adams. E) the stories circulating about Adams's relationship with a slave woman. Ans: C

548. In the election of 1800, the Federalists accused Thomas Jefferson of all of the following except A) having robbed a widow. B) having fathered numerous mulatto children by his own slave women. C) being an atheist. D) supporting high taxes. E) having robbed children of their trust funds. Ans: D

549. In the 1800 election Thomas Jefferson won the state of New York because A) of a reaction against Alexander Hamilton, Jefferson's enemy. B) Aaron Burr used his influence to turn the state to Jefferson. C) of the high taxes passed by the Adams administration. D) Napoleon promised to sell the Louisiana Territory only to Jefferson. E) all of the above. Ans: B

550. The Jeffersonian Democratic-Republicans presented themselves as all of the following except A) believers in a strong central government. B) strict constructionists. C) protectors of agrarian purity. D) believers of political and economic liberty. E) strong supporters of state's rights. Ans: A

551. Thomas Jefferson received the bulk of his support from the A) South and West. B) North. C) large cities. D) East. E) New England. Ans: A

552. In 1800, Thomas Jefferson was chosen president by the A) people. B) Electoral College. C) House of Representatives. D) wealthy. E) business sector. Ans: C

553. Thomas Jefferson's Revolution of 1800 was remarkable in that it A) moved the United States away from its democratic ideals. B) marked the peaceful and orderly transfer of power on the basis of election results accepted by all parties. C) occurred after he left the presidency. D) caused America to do what the British had been doing for a generation regarding the election of a legislative body. E) was in no way a revolution. Ans: B

554. Thomas Jefferson was elected president by the House of Representatives when A) a few Federalists refrained from voting. B) Aaron Burr withdrew from the race. C) Jefferson agreed to appoint John Marshall to the Supreme Court. D) additional Jeffersonians became members of the House. E) the electoral college gave up its responsibility. Ans: A

555. Thomas Jefferson saw his election and his mission as president to include all of the following except A) to return to the original spirit of the revolution. B) restore the republican experiment. C) check the growth of the republican experiment. D) halt the decay of virtue. E) support the establishment of a strong army. Ans: E

556. As president, Thomas Jefferson's stand on several political issues that he had previously championed A) remained unchanged. B) was reversed. C) grew even more rigid. D) compelled him to repeal the Alien and Sedition Acts. E) caused him to reject slavery. Ans: B

557. With Thomas Jefferson's election as president, the Democratic-Republican party A) grew stronger and more unified. B) removed many Federalists from government jobs. C) soon resented its leaders' lavish life-style. D) grew less unified as the Federalist party began to fade and lose power. E) sought to extend the Alien and Sedition Acts to punish their enemies. Ans: D

558. Thomas Jefferson's presidency was characterized by his A) unswerving conformity to Republican party principles. B) rigid attention to formal protocol at White House gatherings. C) moderation in the administration of public policy. D) ruthless use of the patronage power to appoint Republicans to federal offices. E) inability to get legislation passed by Congress. Ans: C

559. On becoming president, Thomas Jefferson and the Republicans in Congress immediately repealed A) the Alien and Sedition Acts. B) the charter of the National Bank. C) the excise tax on whiskey. D) the funding and assumption of the national debt. E) money to fund the naval build-up. Ans: C

560. When it came to the major Federalist economic programs, Thomas Jefferson as president A) left practically all of them intact. B) quickly dismantled them. C) slowly undid everything the Federalists achieved. D) attacked only the Bank of the United States. E) vetoed any new tariffs. Ans: A

561. Thomas Jefferson and his followers opposed John Adams's last-minute appointment of new federal judges mainly because A) the men appointed were of poor quality. B) they believed that the appointments were unconstitutional. C) they did not want a showdown with the Supreme Court. D) it was an attempt by a defeated party to entrench itself in the government. E) these judges were not needed. Ans: D

562. The chief justice who carried out, more than any other federal official, the ideas of Alexander Hamilton concerning a powerful federal government was A) James Madison. B) William Marbury. C) John Marshall. D) Samuel Chase. E) John Jay. Ans: C

563. Before he became Chief Justice of the Supreme Court, John Marshall's service at Valley Forge during the American Revolution convinced him A) to support Thomas Jefferson and his republican principles. B) to give up the life of a soldier and return to law school. C) of the drawbacks of feeble central authority. D) of the futility of opposing Britain. E) all of the above. Ans: C

564. As chief justice of the United States, John Marshall helped to ensure that A) states' rights were protected. B) the programs of Alexander Hamilton were overturned. C) the political and economic systems were based on a strong central government. D) both the Supreme Court and the president could rule a law unconstitutional. E) Aaron Burr was convicted of treason. Ans: C

565. The legal precedent for judicial review was established when A) the House of Representatives impeached Justice Samuel Chase. B) the Supreme Court declared the Judiciary Act of 1789 unconstitutional. C) Congress repealed the Judiciary Act of 1801. D) President Adams appointed several midnight judges to the federal courts. E) the Judiciary Act of 1801 was passed. Ans: B

566. The case of Marbury v. Madison involved the question of who had the right to A) commit the United States to entangling alliances. B) impeach federal officers for high crimes and misdemeanors. C) declare an act of Congress unconstitutional. D) purchase foreign territory for the United States. E) appoint Supreme Court justices. Ans: C

567. John Marshall, as chief justice of the United States, helped to strengthen the judicial branch of government by A) applying Jeffersonian principles in all of his decisions. B) asserting the doctrine of judicial review of congressional legislation. C) overriding presidential vetoes. D) listening carefully to and heeding the advice of lawyers arguing cases before the Supreme Court. E) increasing the number of justices on the Supreme Court. Ans: B

568. Thomas Jefferson's failed attempt to impeach and convict Supreme Court Justice Samuel Chase for high crimes and misdemeanors meant that A) no federal judge could ever be removed from office. B) judicial independence and the separation of powers had been preserved. C) Jefferson's effectiveness as president had been lost. D) an unfortunate precedent had been established. E) Aaron Burr would go free. Ans: B

569. Thomas Jefferson distrusted large standing armies because they A) were usually ineffective in battle. B) always developed a destructive rivalry with the navy. C) could be used to establish a dictatorship. D) all of the above. E) none of the above. Ans: C

570. Thomas Jefferson saw navies as less dangerous than armies because A) they were generally smaller in numbers. B) they had little chance of starting a war. C) they were in less contact with foreign powers. D) they could not march inland and endanger liberties. E) all of the above. Ans: D

571. Thomas Jefferson had strong misgivings about the wisdom of A) states' rights. B) maintaining a large standing army. C) having the presidency and Congress controlled by the same party. D) removing federal judges by the process of impeachment. E) judicial review. Ans: B

572. Thomas Jefferson's first major foreignpolicy decision was to A) purchase Louisiana from France. B) send a naval squadron to the Mediterranean. C) drive the British out of the northwest forts. D) purchase Florida from Spain. E) form an alliance with Spain. Ans: B

573. Thomas Jefferson ceased his opposition to the expansion of the navy when the A) Pasha of Tripoli declared war on the United States. B) U.S. Marine Corps was established. C) mosquito fleet was defeated by the pirates at Tripoli. D) army was disbanded. E) British blockaded the east coast. Ans: A

574. To guard American shores, Thomas Jefferson A) built a fleet of frigates. B) constructed coastal fortifications. C) constructed two hundred tiny gunboats. D) signed a peace treaty with Great Britain. E) enlisted the aid of France. Ans: C

575. Arrange these events in chronological order: (A) Louisiana Purchase, (B) Chesapeake incident, (C) Burr's trial for treason, (D) Embargo Act. A) A, B, D, C B) C, D, A, B C) A, C, B, D D) D, B, C, A E) B, D, C, A Ans: C

576. In order to purchase New Orleans from France, Thomas Jefferson A) threatened to form an alliance with France's enemy, Spain. B) was unwilling to go to war. C) proposed to break away from all alliances to prove our neutrality. D) was willing to use funds from private individuals if Congress would not authorize enough money for the purchase. E) decided to make an alliance with his old enemy, Britain. Ans: E

577. Napoleon chose to sell Louisiana to the United States because A) he had suffered misfortunes in Santo Domingo. B) he hoped that the territory would one day help America to thwart the ambitions of the British. C) he did not want to drive America into the arms of the British. D) yellow fever killed many French troops. E) all of the above. Ans: E

578. Jefferson had authorized American negotiators to purchase only ____________________ from France. A) New Orleans and the Floridas B) New Orleans and St. Louis C) Santo Domingo D) the Missouri River basin E) the entire Louisiana Territory Ans: A

579. Thomas Jefferson was conscience-stricken about the purchase of the Louisiana Territory from France because A) the Federalists supported his action. B) he believed that the purchase was unconstitutional. C) he felt that the purchase was not a fair deal for France. D) war with Spain might occur. E) he feared the British might use it as an exercise to declare war on the United States. Ans: B

580. Lewis and Clark's expedition through the Louisiana Purchase territory yielded all of the following except A) a rich harvest of scientific observations. B) treaties with several Indian nations. C) maps. D) hair-raising adventure stories. E) knowledge of the Indians of the region. Ans: B

581. Lewis and Clark demonstrated the viability of A) travel across the isthmus of Panama. B) an overland trail to the Pacific. C) settlement in the southern portion of the Louisiana territory. D) using Indian guides. E) all of the above. Ans: B

582. After killing Alexander Hamilton in a duel, Aaron Burr A) fled to France. B) fled to England. C) was arrested and found guilty of murder. D) was arrested and found innocent of murder. E) plotted to divide the United States. Ans: E

583. The British policy of impressment was a kind of A) naval blockade. B) economic boycott. C) forced enlistment. D) diplomatic pressure. E) punishment for the United States. Ans: C

584. The British impressed American sailors into the British navy because A) the Americans took the Chesapeake. B) they needed more men. C) Parliament passed a law. D) of the XYZ affair. E) they wanted to punish the United States. Ans: B

585. The Chesapeake incident involved the flagrant use of A) patronage. B) impeachment. C) judicial view. D) impressment. E) naval blockades. Ans: D

586. To deal with British and French violations of America's neutrality, Thomas Jefferson A) declared war on Britain. B) enacted an economic embargo. C) declared war on France. D) did nothing. E) sought trade relations with Spain and Holland. Ans: B

587. Thomas Jefferson's embargo failed for all of the following reasons except that A) he underestimated the determination of the British. B) he underestimated Britain's dependence on American trade. C) Britain produced a bumper grain crop. D) Latin America opened its ports for commerce. E) he miscalculated the difficulty of enforcing it. Ans: B

588. President Jefferson's foreign policy of economic coercion A) underestimated British dependence on American trade. B) adversely affected France's economy more than Britain's. C) stimulated manufacturing in the United States. D) destroyed the Federalist party in New England. E) succeeded in its goal of forcing the British to halt its impressment of American sailors. Ans: C

589. Macon's Bill No. 2 A) forbade American ships from leaving port. B) permitted trade with all nations but promised that if either Britain or France lifted its commercial restrictions on American trade, the United States would stop trading with the other. C) forbade American trade with Britain and France but promised to open trade with either country if it would cease its violations of American neutrality rights. D) repealed the Embargo Act of 1807. E) halted trade with Britain. Ans: B

590. President James Madison made a major foreign-policy mistake when he A) accepted Napoleon's promise to recognize America's rights. B) vetoed Macon's Bill No. 2. C) allied the United States with Britain. D) refused to trust Napoleon. E) declared war on France. Ans: A

591. By 1810, the most insistent demand for a declaration of war against Britain came from A) New England merchants. B) the West and South. C) Federalists. D) the middle Atlantic states. E) southern states. Ans: B

592. The war hawks demanded war with Britain because they wanted to do all of the following except A) wipe out renewed Indian resistance. B) defend American rights. C) gain more territory. D) retaliate for the British burning of Washington, D.C. E) revenge the manhandling of American sailors. Ans: D

593. Of the following, the only argument not put forward by the war hawks as a justification for a declaration of war against Britain was that A) the British armed Indians and incited them to raid frontier settlements. B) British impressment policies were an affront to American nationalism. C) Britain's commercial restrictions had come close to destroying America's profitable New England shipping business. D) British Canada and Spanish Florida were attractive and easily obtainable prizes of war. E) the orders in council stopped the flow of Western farm products to Europe. Ans: C

594. Arrange the following events in chronological order: (A) war hawks enter Congress, (B) declaration of war on Britain, (C) Embargo Act, (D) Battle of Tippecanoe. A) A, B, C, D B) C, A, D, B C) B, C, A, D D) B, A, D, C E) B, C, D, A Ans: B

595. Tecumseh argued that Indians should A) never give control of their land to the whites. B) move west of the Mississippi River. C) not cede control of land to whites unless all Indians agreed. D) exchange traditional buckskin clothing for cloth garments. E) fight as individual tribes and not as a confederacy. Ans: C

596. Native American leader Tecumseh was killed in 1813 at the A) Battle of Tippecanoe. B) Battle of the Thames. C) Battle of Horseshoe Bend. D) Battle of New Orleans. E) Battle of Fallen Timbers. Ans: B

597. The battle of Tippecanoe resulted in A) defeat of the British by the hands of the Indian confederacy. B) a Shawnee loss and a Creek victory. C) a declaration of war by the United States on Great Britain. D) the expulsion of the British from Florida. E) the death of the dream of an Indian confederacy. Ans: E

598. In 1812, James Madison turned to war A) to help him win re-election. B) due to his hatred of Great Britain. C) to fulfill alliance obligations with France. D) to fulfill alliance obligations with Spain. E) to restore confidence in the republican experiment. Ans: E

599. Seafaring New England opposed the War of 1812 because of all of the following except A) the Northeast Federalists sympathized with England. B) it resented the Republican's sympathy with Napoleon. C) Federalists opposed the acquisition of Canada. D) it could result in more agrarian states. E) their strong trade ties with France. Ans: E

600. Once begun, the War of 1812 was supported strongly by A) practically all Americans. B) New England and the seaboard states. C) very few people. D) the West and South. E) Native Americans. Ans: D

601. Federalists opposed the acquisition of Canada because A) there were too many French there. B) Canadian business would prove too competitive. C) it was too agrarian and would give more votes to the Democratic-Republicans. D) they believed that the Canadians could never become Americanized. E) too many Indians lived there. Ans: C

602. During the War of 1812, the New England states A) supported the United States' war effort. B) lent more money and sent more food to the British army than to the American army. C) gave no support to either the Americans or the British. D) allowed their militias to fight wherever the federal government requested. E) declared their independence from the United States. Ans: B

Você também pode gostar